LSAT and Law School Admissions Forum

Get expert LSAT preparation and law school admissions advice from PowerScore Test Preparation.

User avatar
 Dave Killoran
PowerScore Staff
  • PowerScore Staff
  • Posts: 5862
  • Joined: Mar 25, 2011
|
#49529
Complete Question Explanation
(The complete setup for this game can be found here: lsat/viewtopic.php?t=18147)

The correct answer choice is (E)

Answer choice (A): The 2-2-1-1 numerical distribution in this game forces F and H to review the same movie, and so no other reviewer aside from H can review a movie with F. This answer choice attempts to pair F and G, and is thus incorrect.

Answer choice (B): The 2-2-1-1 numerical distribution in this game forces F and H to review the same movie, and so no other reviewer aside from H can review a movie with F. This answer choice attempts to pair F and L, and is thus incorrect.

Answer choice (C): G must review M, and J must review W, and so the two can never review the same movie.

Answer choice (D): If K reviews W and L reviews M, then there is no solution to the game that would fit the 2-2-1-1 distribution. Because J must review M or W, either a 3-2-1 or a 2-2-2 distribution results, and in neither of those two distributions are all four movies reviewed. Thus, this answer choice cannot be true and is incorrect.

Answer choice (E): This is the correct answer choice. The hypothetical produced in question #7 helps show that this answer can be true.

Get the most out of your LSAT Prep Plus subscription.

Analyze and track your performance with our Testing and Analytics Package.